La ricerca ha trovato 30 risultati

da lucaboss98
14 set 2014, 13:01
Forum: Algebra
Argomento: Funzionale giocosa. . .
Risposte: 10
Visite : 4312

Re: Funzionale giocosa. . .

Bisogna dimostrare che $ \beta = \gamma $ , allora..
da lucaboss98
14 set 2014, 01:15
Forum: Algebra
Argomento: Funzionale giocosa. . .
Risposte: 10
Visite : 4312

Re: Funzionale giocosa. . .

Fissato $x_0$ definisco la successione $x_{m+1} = f(x_m) $ . Si ha quindi , sostituendo $x_0, x_1 , \ldots $ al posto di $n$ (si può fare perchè $x_m \in \mathbb{N} \forall m \in \mathbb{N} $ ) \begin{equation} x_{m+3} = 3x_{m+2} - 6x_{m+1} + 4x_m + 2001 \end{equation} Sia $ x_m = a_m + b_m $ dove $...
da lucaboss98
12 set 2014, 20:13
Forum: Teoria dei Numeri
Argomento: Mexico 2013
Risposte: 2
Visite : 4100

Re: Mexico 2013

Divido in due casi: 1) se $ b=1 $ si ha $ p_a - 2 \mid 2a - 2 $ da cui $ 2a \geq p_a $. Ora, visto che $a \geq 3 $ , si ha che $ p_{a+1} \geq p_a + 2 $. Dimostro per induzione su $a$ che $2a < p_a $ per $ a \geq 5 $. Passo Base: $ a=5 $ si ha $ 10 < 11 $ Passo Induttivo: Supposto che $ 2a < p_a $ , ...
da lucaboss98
10 set 2014, 16:05
Forum: Olimpiadi della matematica
Argomento: Senior 2014
Risposte: 146
Visite : 60549

Re: Senior 2014

"Ma a Napoli non c'è il mare"
"Ad Asti c'è un fiume"
Le perle di saggezza...
da lucaboss98
09 set 2014, 18:17
Forum: Olimpiadi della matematica
Argomento: Senior 2014
Risposte: 146
Visite : 60549

Re: Senior 2014

matpro98 ha scritto: -due sorelle quasi imbattibili a biliardino.
Girano leggende metropolitane, del tipo che qualcuno sia riuscito a batterle.. :lol:
Drago96 ha scritto: - #LucaAlleBMO!
State esagerando...
scambret ha scritto: - gli scherzi fatti a una neo-spesata
Rigorosamente sul mio letto mentre io non c'ero... :(
da lucaboss98
26 giu 2014, 17:21
Forum: Olimpiadi della matematica
Argomento: Senior 2014
Risposte: 146
Visite : 60549

Re: Senior 2014

Grazie mille, EvaristeG! :D
da lucaboss98
26 giu 2014, 00:03
Forum: Teoria dei Numeri
Argomento: Disuguaglianza $\phi$ga
Risposte: 6
Visite : 5521

Re: Disuguaglianza $\phi$ga

EDIT: errore :oops: :oops:
da lucaboss98
23 giu 2014, 13:11
Forum: Olimpiadi della matematica
Argomento: Senior 2014
Risposte: 146
Visite : 60549

Re: Senior 2014

Buongiorno, scusate, sapreste se nell' esericizio N2 del preIMO 2013 i primi sono distinti fra loro?
da lucaboss98
16 giu 2014, 19:56
Forum: Olimpiadi della matematica
Argomento: Senior 2014
Risposte: 146
Visite : 60549

Re: Senior 2014

Scusate le mie continue domande :lol: :lol: ma Bunching e Schur si possono usare?
da lucaboss98
15 giu 2014, 20:04
Forum: Olimpiadi della matematica
Argomento: Senior 2014
Risposte: 146
Visite : 60549

Re: Senior 2014

Anche che il simmetrico dell'ortocentro rispetto al punto medio di un lato appartiene alla circoscritta si può dare per scontato?
da lucaboss98
14 giu 2014, 01:34
Forum: Olimpiadi della matematica
Argomento: Senior 2014
Risposte: 146
Visite : 60549

Re: Senior 2014

Chi ha i problemi della mattina può usare i polinomi di Lagrange, vero?
da lucaboss98
13 giu 2014, 17:38
Forum: Algebra
Argomento: $(x^3+y^3+z^3)^2+3(xyz)^2$
Risposte: 2
Visite : 2129

Re: $(x^3+y^3+z^3)^2+3(xyz)^2$

EDIT: errore nel Bunching
da lucaboss98
19 mag 2014, 18:54
Forum: Algebra
Argomento: Problema a squadre
Risposte: 10
Visite : 5869

Re: Problema a squadre

Ma non è $ f(n) = 50n(n+1) $ ? Se è così: Allora f(14)< g(14) e f(13)>g(13) quindi $ n=14 $ da cui $ g(14) - f(14) = 2^{14} - 1 - 50 \cdot 14 \cdot 15 = 5883 $ Se scrivi 2 elevato a n allora è 50(n-1)(n) No, primo mese: $ g(1) = 1 = 2^1 - 1 $ e $ f(1) = 100 = 50 \cdot 1 \cdot 2 $ Secondo mese: $ g(...
da lucaboss98
17 mag 2014, 16:54
Forum: Algebra
Argomento: Problema a squadre
Risposte: 10
Visite : 5869

Re: Problema a squadre

Ma non è $ f(n) = 50n(n+1) $ ?
Se è così:
Allora $ f(14)< g(14) $ e $ f(13)>g(13) $
quindi $ n=14 $
da cui $ g(14) - f(14) = 2^{14} - 1 - 50 \cdot 14 \cdot 15 = 5883 $
da lucaboss98
16 mag 2014, 00:25
Forum: Teoria dei Numeri
Argomento: Problema n.3 Cesenatico 2014
Risposte: 10
Visite : 5818

Re: Problema n.3 Cesenatico 2014

Ma il punto (b) ti chiede di dimostrare che per ogni $ k $ esiste un $ D_n $, mi pare tu l'abbia fatto solo per $ k=0,1 $. Comunque per il punto (a) rileggendolo può andare anche perché il caso che non hai trattato, $ D_n=1 $ basta ricordarsi che $ 1=3^0 $ comunque credo vada specificato.